LSAT and Law School Admissions Forum

Get expert LSAT preparation and law school admissions advice from PowerScore Test Preparation.

 Administrator
PowerScore Staff
  • PowerScore Staff
  • Posts: 8917
  • Joined: Feb 02, 2011
|
#26481
Question Line Reference
(See the complete passage discussion here: lsat/viewtopic.php?t=11045)

The correct answer choice is (D)

The justification for the correct answer can be found on:

(Lines 1-7; 50-60)

With an open-ended Must Be True question like this one, there is no way to prephrase the answer choice, so we must seek the choice confirmed by the information provided in the stimulus. Only answer choice (D) can reasonable be inferred from the information in the passage, based especially on the first paragraph and the opening sentence of the last paragraph.
 cindyhylee87
  • Posts: 29
  • Joined: May 21, 2017
|
#35894
Hi,

I had two contenders after reviewing all 5 answer choices. I finally chose (A) since line 1-3 was saying that most authoritarian rulers undertake democratic reforms...and line 55 and onward was saying that they would realize that democratic reform is their only hope to further maintain some power.

Not so sure what I was missing to select the correct answer.

Thanks,
Cindy
 Adam Tyson
PowerScore Staff
  • PowerScore Staff
  • Posts: 5153
  • Joined: Apr 14, 2011
|
#36036
Hey Cindy, try re-reading answer A and comparing it to the very first sentence in the passage. "They would do it anyway" (paraphrasing here) is in direct conflict with the author's view that these authoritarian rulers make democratic changes only because they see that they are necessary if they want to stay in power. That's what I would call "pressured to do so."

There's nothing in the passage to suggest that authoritarian rules would make these changes without such pressure, and that makes answer A an opposite answer and incorrect. "Pressure" here doesn't necessarily mean direct pressure from people, like a trusted adviser or wealthy upper-class elites telling the ruler to make the changes or else lose his support, but can include pressure in the form of societal changes that dictate their behavior.

Let us know if that makes sense to you on second look? Keep at it!
 cindyhylee87
  • Posts: 29
  • Joined: May 21, 2017
|
#36048
Thank you Adam. It does make sense now :)

Cindy
 rtemkin23
  • Posts: 4
  • Joined: Feb 14, 2020
|
#73863
Hi there,

I also chose answer choice A, and while I guess I see why A is wrong (although I do feel like it's a stretch to say that authoritarian leaders who make calculated decisions to preserve their own power is still considered a form of pressure here), I'm very much at a loss for why the correct answer is D.

First of all the wording is definitely confusing, but I ultimately take the double negative "does not condone" to mean that absolute power can't be maintained if the society approves of, or feels neutral about, the authoritarian regime. My confusion here is that, yes, I see that this answer explains the author's argument, but the evidence is based on language of possibility: "authoritarian rule tends" and "all three types can be accelerated by the authoritarian regime's initial success. My question is, why is this inference based on weak unprovable language a better answer over one where the definition of "pressure" is much loser?

Thanks!
 Jeremy Press
PowerScore Staff
  • PowerScore Staff
  • Posts: 1000
  • Joined: Jun 12, 2017
|
#73894
Hi rtemkin,

I think part of the confusion you're feeling might be from your reading of the phrase "does not condone," which actually is just a negative phrase (not a double negative). To condone something means to approve it (a positive notion). When the answer choice says, "if the society does not condone the regime," it's just saying if the society does not approve that regime (so the "not" here actually does render the idea negative, rather than it becoming positive as it would in a true double negative).

Another problem might be where you're looking for the answer's support. The source for answer choice D is actually not the end of the passage. It's back at the end of the first paragraph and the beginning of the second paragraph, where the author writes (with certainty) that "[authoritarian rulers] foresee or recognize that certain changes and mobilizations in civil society make it impossible for them to hold on indefinitely to absolute power," and then goes on to write that "[t]hree major types of changes can contribute to a society’s no longer condoning the continuation of authoritarian rule." These sentences leave us two strong impressions: first, the author thinks that certain societal changes (whatever their combination turns out to be in a particular place) make loss of authoritarian power inevitable, and second, the author thinks that loss of authoritarian power is coextensive with "society's no longer condoning the continuation of authoritarian rule" (because in each of these two sentences the causal driver is societal change). The author's certainty about it being "impossible for them to hold on indefinitely" is what validates the certainty of answer choice D.

I hope this helps!

Jeremy
 rtemkin23
  • Posts: 4
  • Joined: Feb 14, 2020
|
#73933
Hi Jeremy-yes, this helps a ton. Can't believe I messed up the definition of condone....Thank you so much!

Get the most out of your LSAT Prep Plus subscription.

Analyze and track your performance with our Testing and Analytics Package.